Microsoft Math Solver
Solve
Practice
Download
Solve
Practice
Topics
Pre-Algebra
Mean
Mode
Greatest Common Factor
Least Common Multiple
Order of Operations
Fractions
Mixed Fractions
Prime Factorization
Exponents
Radicals
Algebra
Combine Like Terms
Solve for a Variable
Factor
Expand
Evaluate Fractions
Linear Equations
Quadratic Equations
Inequalities
Systems of Equations
Matrices
Trigonometry
Simplify
Evaluate
Graphs
Solve Equations
Calculus
Derivatives
Integrals
Limits
Algebra Calculator
Trigonometry Calculator
Calculus Calculator
Matrix Calculator
Download
Topics
Pre-Algebra
Mean
Mode
Greatest Common Factor
Least Common Multiple
Order of Operations
Fractions
Mixed Fractions
Prime Factorization
Exponents
Radicals
Algebra
Combine Like Terms
Solve for a Variable
Factor
Expand
Evaluate Fractions
Linear Equations
Quadratic Equations
Inequalities
Systems of Equations
Matrices
Trigonometry
Simplify
Evaluate
Graphs
Solve Equations
Calculus
Derivatives
Integrals
Limits
Algebra Calculator
Trigonometry Calculator
Calculus Calculator
Matrix Calculator
Solve
algebra
trigonometry
statistics
calculus
matrices
variables
list
Differentiate w.r.t. θ
6\theta ^{5}
View solution steps
Steps Using Definition of a Derivative
\theta ^ { 6 } =
The derivative of ax^{n} is nax^{n-1}.
6\theta ^{6-1}
Subtract 1 from 6.
6\theta ^{5}
Evaluate
\theta ^{6}
Graph
Quiz
Arithmetic
5 problems similar to:
\theta ^ { 6 } =
Similar Problems from Web Search
Using the MSE criterion, which is a better estimator for \Theta^2?
https://math.stackexchange.com/q/191043
I just give an answer as the persons visiting the question shouldnt go through all the conversations. Without imposing another conditions on the parameter \Theta or the types of these two ...
A question about modular forms in SAGE
https://math.stackexchange.com/questions/135472/a-question-about-modular-forms-in-sage
Thank you for reading my book. I am not an expert in Sage, but I can tell you what I had in mind when I wrote that problem. The goal of the exercise is to work with \Theta^6 replicating what is done ...
Containing an open set = being an open set?
https://math.stackexchange.com/questions/2184858/containing-an-open-set-being-an-open-set
Saying \Theta' is open is much stronger as saying that it contains some open rectangles. It says that for all \theta \in \Theta', there is an open rectangle R with \theta \in R \subset \Theta' ...
Find asymptotic variance MLE heavy tailed distribution
https://math.stackexchange.com/q/1244514
Your MLE for \theta does not seem correct. If \boldsymbol x = (x_1, \ldots, x_n) is an iid sample drawn from a X \sim \operatorname{Pareto}(1,\theta) distribution with density f_X(x) = \theta x^{-\theta-1} \mathbb{1}(x > 1) ...
Extensions of Ramanujan's Cos/Cosh Identity
https://math.stackexchange.com/questions/517409/extensions-of-ramanujans-cos-cosh-identity
It will be helpful to start from an explanation of the origin and the proof of the Ramanujan identity. These are hidden (not very deeply) in the theory of elliptic functions. Indeed, Jacobi elliptic ...
Expectation maximization when support of likelihood is parameterized
https://stats.stackexchange.com/q/318089
I think g(\theta;\theta^0) may have a mistake. If 0 < \theta_2 < 4, then g(\theta;\theta^0)=\frac{1}{4} \int_0^{\theta_2} \ln \frac{1}{\theta_2} dx_{32} = -\frac{\theta_2}{4} \ln \theta_2. ...
More Items
Share
Copy
Copied to clipboard
6\theta ^{6-1}
The derivative of ax^{n} is nax^{n-1}.
6\theta ^{5}
Subtract 1 from 6.
Examples
Quadratic equation
{ x } ^ { 2 } - 4 x - 5 = 0
Trigonometry
4 \sin \theta \cos \theta = 2 \sin \theta
Linear equation
y = 3x + 4
Arithmetic
699 * 533
Matrix
\left[ \begin{array} { l l } { 2 } & { 3 } \\ { 5 } & { 4 } \end{array} \right] \left[ \begin{array} { l l l } { 2 } & { 0 } & { 3 } \\ { -1 } & { 1 } & { 5 } \end{array} \right]
Simultaneous equation
\left. \begin{cases} { 8x+2y = 46 } \\ { 7x+3y = 47 } \end{cases} \right.
Differentiation
\frac { d } { d x } \frac { ( 3 x ^ { 2 } - 2 ) } { ( x - 5 ) }
Integration
\int _ { 0 } ^ { 1 } x e ^ { - x ^ { 2 } } d x
Limits
\lim _{x \rightarrow-3} \frac{x^{2}-9}{x^{2}+2 x-3}
Back to top